Help asap

What is the end behavior of the following graph

Help Asap What Is The End Behavior Of The Following Graph

Answers

Answer 1

Using limits, it is found that the end behavior of the graph is given as follows:

It rises to the left, and stays constant at y = -4 to the right.

What is the end behavior of a function f(x)?

It is given by the limits of f(x) as x goes to infinity.

In this problem, the function is given by:

[tex]f(x) = 4\left(\frac{2}{5}\right)^{x + 3} - 4[/tex]

Hence:

[tex]\lim_{x \rightarrow -\infty} f(x) = \lim_{x \rightarrow -\infty} 4\left(\frac{2}{5}\right)^{x + 3} - 4 = 4\left(\frac{5}{2}\right)^{\infty + 3} - 4 = \infty - 4 = \infty[/tex]

[tex]\lim_{x \rightarrow \infty} f(x) = \lim_{x \rightarrow -\infty} 4\left(\frac{2}{5}\right)^{x + 3} - 4 = 4\left(\frac{2}{5}\right)^{\infty + 3} - 4 = 0 - 4 = -4[/tex]

Hence:

It rises to the left, and stays constant at y = -4 to the right.

More can be learned about limits and end behavior at https://brainly.com/question/22026723

#SPJ1


Related Questions

Solve the following equation, and check your solution. Be sure to show all work in order to receive full credit.

Solve -17 + n/5 = 33.

Answers

Answer:

n = 250

See solution and check blow.

Step-by-step explanation:

Solution:

-17 + n/5 = 33

Add 17 to both sides.

-17 + 17 + n/5 = 33 + 17

n/5 = 50

Multiply both sides by 5.

n/5 × 5 = 50 × 5

n = 250

Check:

-17 + n/5 = 33

Since we found out that n = 250, substitute 250 for n in the equation.

-17 + 250/5 = 33

Simplify the left side. Divide 250 by 5.

-17 + 50 = 33

Add on the left side.

33 = 33

Since 33 = 33 is a true statement, the solution n = 250 is correct.

A square with side lengths 12 feet. 4 2 feet by 2 feet squares are cut out of each corner of the square.
Tariq designed the pool shown. The owner of the pool has one square cover to use. Find the area of the space that needs to be covered. (The four corners are squares.)

The area that needs to be covered is

ft2.

Answers

Answer:

132ft^2

Step-by-step explanation:

can someone please help and explain its due in a littleee

Answers

Answer:

Step-by-step explanation:

I need to know which ones are parallel and which ones are perpendicular and the steps thank youu

Answers

Answer:

If both equations have the same slope, they are said to be parallel.

If an equation is the reciprocal of another equation (meaning flipped denominator and numerator as well as flipped +/- sign) they are said to be Perpendicular.

Hope it helps!

Parallel lines will have the same slope! Perpendicular lines have opposite reciprocal slopes (opposite signs and reciprocal)

y = mx + b where m is the slope

The first and last equations have 2 as their m value and therefore are PARALLEL

Since the second equation has -1/2 which is the opposite reciprocal of 2, it’s PERPENDICULAR to the other two lines :)

The cost of the toy is expressed by the function y = 3x + 5, where x is the number of toys. If Peter bought 8 toys, find the cost of the toys.

Answers

Answer: If Peter bought 8 toys, the cost of the toys will be 29.

From the equation, y = 3x + 5, they have given the value of x as eight. So if we put eight in the equation and solve it, we will get the value of y. I believe there are some errors in the question as it doesn't specify that y is the cost. Also cost requires some sort of unit, which is not given in the context. I can't really tell if it is dollars or rubles or pounds etc. So I just gave it as the way it is. Since no other clues are given, this is most likely it.

y = 3x + 5

y = 3(8) + 5

y = 24 + 5

y = 29


∩_∩
(„• ֊ •„)♡
┏━∪∪━━━━┓
   hope it helped
┗━━━━━━━┛

In ABC, CT is a median. What is the measure of AB?

Fill in the blank by enter in just a number for your answer.

Answers

Answer:

AB = 102

Step-by-step explanation:

the median CT bisects the side AB at T , then

BT = AT , that is

7x + 9 = 5x + 21 ( subtract 5x from both sides )

2x + 9 = 21 ( subtract 9 from both sides )

2x = 12 ( divide both sides by 2 )

x = 6

then

AB = 5x + 21 + 7x + 9 = 12x + 30 = 12(6) + 30 = 72 + 30 = 102

The measure of AB is 102.

How to estimate the measure of AB?

The median CT bisects the side AB at T, then

BT = AT

7x + 9 = 5x + 21

subtracting 5x from both sides of the equation, we get

2x + 9 = 21

subtract 9 from both sides, and we get

2x = 12

Divide both sides by 2, we get

x = 6

then, AB = 5x + 21 + 7x + 9

AB = 12x + 30

Substituting the value of x,

12x + 30 = 12(6) + 30

= 72 + 30

AB = 102

Therefore, the measure of AB is 102.

To learn more about triangles

https://brainly.com/question/11070154

#SPJ2

Mr. Sanchez’s class sold fruit pies for $1.73 each and Mr. Kelly’s class sold bottles of fruit juice for $1.44 each. Together, the classes sold 83 items and earned $130.83 for their school. Write and solve a system of equations that models this problem. Show all your work.

Answers

Answer:

p = 39       f = 44    

Step-by-step explanation:

p = $ 1.73

f = $ 1.44

equation

p + f = 83

1.73 p + 1.44 (83 - p) = $ 130.83

p = 39 (amount of times fruit pies were sold).

Therefore,

p = 39

f = 44

If 4y = 3x - 1, then x = ?

Answers

Answer:

[tex]x = \frac{4y + 1}{3} [/tex]

The goal is to isolate the X value. So the first step would be to add 1 to both sides, making the equation 4y + 1 = 3x.

Then you divide both sides by 3, making the equation (4y + 1)/3 = x

If you have a Y value then you'd plug it in for Y and solve for x but since there isn't one provided you're done.

can someone please help me?

Answers

A slope of a line.

We have the equation of a line:

[tex]y=-\dfrac{5}{4}x-1[/tex]

The formula of a slope of a line passes through points A, B:

[tex]A(x_A,\ y_A),\ B(x_B,\ y_B)\\\\m=\dfrac{y_B-y_A}{x_B-x_A}[/tex]

Complete the table:

Put the values of x to the equation of a line and calculate the value of y:

[tex]x=0\\\\y=-\dfrac{5}{4}\cdot0-1\\\\y=-1\\\\\\x=4\\\\y=-\dfrac{5}{4}\cdot4-1\\\\y=-6[/tex]

Substitute to the formula of a slope:

[tex]m=\dfrac{-6-(-1)}{4-0}\\\\m=\dfrac{-6+1}{4}\\\\\huge\boxed{m=-\dfrac{5}{4}}[/tex]

f(x) = |x – 3| + 6 where is the vertex of the graph

Answers

Answer: (3,6)

Rewrite in vertex form and use this form to find the vertex (h,k)

Answer:

(3,6)

Step-by-step explanation:

f(x) = |x| is a V-shaped graph with the vertex at (0,0). It is called a parent graph, because its super basic.

f(x) = |x - 3| + 6

has had two changes made to it. The "- 3" that's in there close to the x slides the graph over to the right 3 units. This is kind of the opposite of what you might think. Minus3 shifts right. (Plus3 would shift left)

The +6 tacked on to the end shifts the whole graph UP 6.

That slides the graph RIGHT3 and UP6. So the vertex lands at (3,6). See image.

Question 3 of 10
What else would need to be congruent to show that AABC= AXYZ by ASA?
B
A
C Z
Y
X
Given:

Answers

I’m pretty sure that is should be y but I’m not too sure about it

PLEASE ANSWER ASAP!!!!!!
What is the component form of the vector represented by 2a when a⃗ =<−2,3>?

Enter the answer in the boxes.

2a= <__,__>

Answers

The complete question is

"What is the component form of the vector represented by 2a when a =⟨−2,3⟩?"

The Component form of the vector represented by 2a is (-4,6)

What is the component form of a vector?

The component form of a vector is represented by (x, y) that has gone x units left to the origin and y units up from the origin.

Given, a=(-2,3)

By using scalar multiplication,

2a = (2(-2), 2(3))

2a = (-4,6).

Therefore the component form of 2a=(-4,6)

For more on vectors visit-

https://brainly.com/question/16810858

#SPJ1

Can someone please help me with this

Answers

Answer:

The last figure.

Answer:

Figure 1, 3 and 6

Point symmetry is when two figures or (equal) parts of a figure lies at a same distance but in opposite direction from a certain point.

What is the value of the expression below?

27−−√3
A
3

B
9

C
24

D
81

Answers

Answer:

I don't see a correct answer.  See below.

Step-by-step explanation:

Thanks for clarifying the question.  But if 27√3 is the correct expression, I don't see an answer option that fits.  

√3 = 1.732

27*(1.732) = 46.77

But all the answer options are whole numbers.  The presence of√3 makes it difficult to find an expression that would result in a whole number.  Expressions such as:

27/√9 = 9, or

√27/√3 = 3

would be more consistent with producing a whole number result.

I hope this helps.

What is the range of the function shown
in the graph?

Answers

Choice B. Y value max is 8 and it goes down from there, so 8 is the greatest value it can be.

If 2 inches is 5 centimeters, which of following is correct?

Answers

Unit conversion is a way of converting some common units into another without changing their real value. The correct option is D.

What is unit conversion?

Unit conversion is a way of converting some common units into another without changing their real value. for, example, 1 centimeter is equal to 10 mm, though the real measurement is still the same the units and numerical values have been changed.


The complete question is mentioned in the below image.

Given that 2 inches are 5 centimeters. Therefore, the ratio of an inch to centimeters are,

2 inches = 5 centimeters

1 inch = 5/2 centimetes

Now, if using the ratio, the measurements are converted then,

A.) 7 inches = 7 × (5/2)centimeters = 17.5 centimetes

B.) 5 inches = 5 × (5/2)centimeters = 12.5 centimetes

C.) 8 inches = 8 × (5/2)centimeters = 20 centimetes

D.) 3 inches = 3 × (5/2)centimeters = 7.5 centimetes

As it can be observed that the correct statement is D If 2 inches is 5 centimeters, then 3 inches is 7.5 centimeters. Hence, the correct option is D.

Learn more about Units conversion:

https://brainly.com/question/4736731

#SPJ1

helppppppppp markingbrainliest

Answers

a)

x×3+4=22

b)

x×3+4=22

x×3=22-4

x×3=18

x=18÷3

x=6

Hope it helps

Answer:

x=6

Step-by-step explanation:

your equation would be X * 3 + 4 = 22

to solve you subtract four from both sides then divide three by both sides because you do the opposite of the sign thats there and you get x= 6! i hope this helped

The equation y = 180 + 5x represents the amount a school club will pay a T-shirt company for custom T-shirts, where x represents the number of T-shirts purchased and y represents the total cost in dollars. The equation y = 10x represents the club’s income from selling the T-shirts, where y represents the money earned in dollars and x represents the number of T-shirts sold. Assuming that the club members sell all the T-shirts they purchase, when will the club make a profit? The following graph represents this situation:

Answers

Answer:

36 TSHIRTS is the number for minimum profit

Step-by-step explanation:

Y = Y

180 + 5X = 10X

180 = 10X - 5X

180 = 5X

X = 180/5

X = 36

Question 3 of 10
Which choice represents the simplified exponential expression?
(12-4)8
OA. 12-32
B. 12-12
O C. 12
OD. 124

Answers

The correct value that equates to this expression is 12‐³². Letter A

.

To solve this expression, just: eliminate the parentheses and multiply the exponents among themselves;[tex] \boxed{ \large \sf (a {}^{n} ) {}^{m} \rightarrow a {}^{n \times m} } \\ \\ [/tex]

Resolution

[tex]{ = \large \sf (12{}^{-4} ) {}^{8} } [/tex]

[tex]{ = \large \sf 12{}^{-4 \times 8} } [/tex]

[tex] \pink{ \boxed{ = \large \sf 12{}^{-32} } } \\ [/tex]

Therefore, the answer will be 12‐³²

What is the area of this trapezoid?

340 cm²

427.5 cm²

440 cm²

487.5 cm²

Answers

Answer:

440cm²

Step-by-step explanation:

*Area= (a+b)/2×h

(19+25)/2*20

= _ * 440cm². *_

Answer:

440cm²

Step-by-step explanation:

I believe its right

If x is increased by 25% then, by what % is x² increased

Answers

By analytical methods and algebraic handling and on the basis that x is increased by 25 %, we conclude that the variable x² is increased by 18.75 %.

How to determine the percentage increase rate

In this case we have a linear variable x, whose increase is described by the following expression:

y = x · (1 + r/100)     (1)

Where r is the increase rate.

If we square (1), then we find that:

y² = x² · (1 + r/100)² = (1 + r'/100)     (2)

Where r' is the equivalent increase rate.

(1 + r/50 + r²/10000) = (1 + r'/100)

r'/100 = r/50 + r²/10000

r' = r/2 + r²/100

If we know that r = 25, then the equivalent increase rate is:

r' = 25/2 + 25²/100

r' = 18.75

By analytical methods and algebraic handling and on the basis that x is increased by 25 %, we conclude that the variable x² is increased by 18.75 %.

To learn more on percentages: https://brainly.com/question/14979505

#SPJ1

Peter's father told Peter to buy four-tenths of a pound of chili powder. Each package is marked with its weight. What package should Peter buy?

Answers

Peter should buy 0.4 pound weight package

A percentage is a figure or ratio stated as a fraction of 100 in mathematics. Although the abbreviations "pct," "pct," and occasionally "pc" are also used, the percent sign, " percent ", is frequently used to signify it. A % is a number without dimensions and without a measurement system.

Given Peter's father told Peter to buy four-tenths of a pound of chili powder and Each package is marked with its weight.

We have to find which weight package should Peter buy

The four-tenths of a pound will be 40 % of one pound which is 0.4 pound. So peter should buy 0.4 pound weight package.

Hence Peter should buy 0.4 pound weight package

Learn more about percentage here:

https://brainly.com/question/843074

#SPJ10

Are survival rates independent of gender within similar passenger classes ?

Answers

Answer:

yes

Step-by-step explanation:

Women had a much higher chance of survival — regardless of what class they were in — then men did. Of the 466 women on board, 339 survived. Of the 843 men on board, only 161 survived — a measly 19% compared to the 73% of women who made it safely back to shore.

please answer I've got 10 mins
100 POINTS FOR BRAINLIEST

Determine if the two expressions are equivalent and explain your reasoning.
3(4n) + 2 + 6n and 13n + 2

Answers

Answer:

3(4n) + 2 + 6n

12n + 2 + 6n

12n + 6n + 2

18n + 2 is not equal to 13n + 2

hence both the expression are not equivalent

x = √3 + √2/ √3 - √2
y = √3 - √2/ √3 + √2
Find x² + y²

Answers

Answer: 98

Given following:

[tex]\Longrightarrow \sf x = \dfrac{\sqrt{3} +\sqrt{2} }{\sqrt{3} - \sqrt{2} }[/tex]

[tex]\Longrightarrow \sf y = \dfrac{\sqrt{3} -\sqrt{2} }{\sqrt{3} + \sqrt{2} }[/tex]

Solve for x² + y²:

[tex]\rightarrow \sf \left(\:\dfrac{\sqrt{3}\:+\sqrt{2}\:\:}{\sqrt{3}\:-\:\sqrt{2}\:}\right)^2\:+\:\left(\dfrac{\sqrt{3}\:\:-\sqrt{2}\:\:}{\sqrt{3}\:+\:\sqrt{2}\:\:}\:\right)^2[/tex]

simplify following

[tex]\rightarrow \sf 49+20\sqrt{6}+49-20\sqrt{6}[/tex]

add/subtract similar terms

[tex]\rightarrow \sf 98[/tex]

answer it please!!!!

Answers

Answer:

the answer is B  

Step-by-step explanation:

I used photo math it shows the correct answer

Answer:

2nd one (B)

Step-by-step explanation:

The lines shown below are perpendicular. If the green line has a slope of
1/7 what is the slope of the red line?
A.7
B.1/7
C.-7
D. -1/7

Answers

Answer:

c: -7

Step-by-step explanation:

the slope of a perpendicular line is the negative reciprocal of the original line

a way to find this is to flip the numerator and the denominator of a slope and make it negative

1/7 --> 7 --> -7

that would be your slope

Explain some acceptable reasons for proofs?

Answers

Geometric proofs are a list of Statements and Reasons used to prove that a given mathematical concept or idea is true. Statements are claims about a geometric problem that cannot be proven true until backed by a mathematical Reason. Reasons are pieces of evidence that support a Statement.

Which is a nonlinear relationship

Answers

Answer:

D. 4

Explanation:

Linear relationship refers to relationships which give straight lines when graphed. All the segments, except segment 4, show straight-line, linear relationships.

As section 4 is curved, it show nonlinear relationship.

PLEASE ANSWER ASAP!!!!!!
The function f(x)=−6sin(7/3x+1/6)−5 models the average rate of change in temperature of a substance monitored in an experiment. In the function, x represents the number of minutes since the commencement of the experiment and the temperature of the substance is measured in degrees Fahrenheit.

Over what range of temperatures does the average rate of change in temperature fall?

Enter a value in each box to correctly complete the statement.

The range in the average rate of change in temperature of the substance is from a low
temperature of ____ F to a high of ________ F

Answers

The range in the average rate of change in temperature of the substance is from a low temperature of 1 F to a high of -11 F.

What is a formula for Fahrenheit?

The conversion formula for a temperature that is expressed on the Celsius (°C) scale to its Fahrenheit (°F) ;

°F = (9/5 × °C) + 32.

Given function:

f(x)= -6 sin(7/3 x+ 1/6) -5

The function will be maximum at the 7/3 x +1/6= 3π/2

So, the maximum temperature will be

= -6 sin (3π/2) -5

= 6 -5

= 1 F

The function will be minimum at the 7/3 x +1/6= π/2

Therefore, the maximum temperature will be

= -6 sin (π/2) - 5

= -6 -5

= -11 F

Learn more about this concept here:

brainly.com/question/26172293

#SPJ1

Other Questions
Lydia writes the equation below with a missing value.y = 5 x minus boxShe puts a value in the box and says that the equation represents a direct variation. Which explains whether the equation could represent a direct variation?If she puts 0 in the box she would have a direct variation.If she puts 5 in the box she would have a direct variation.The equation is not of the form y = k x, so it cannot represent a direct variation.The equation has a minus sign, so it cannot represent a direct variation. Please help! Multiply the following two numbers in scientific notation. Which sentence uses exact words and sensory details to help the reader picture whats happening? A. The day I learned to water ski was one of the best days of my life. B. I loved the feel of cool water spraying against my face. C. Someday, I want to learn to snowboard. D. I hope I can go water skiing again, soon! Calcular la altura, h de un arbol sabiendo que, si nos situamos 8 metros de la base del tronco, vemos la parte superior de su copa en un angulo de 36.87* If m < 0 and b > 0, the graph of y = mx + b does not pass through which quadrant?Quadrant IQuadrant IIQuadrant IIIQuadrant IV Which explains to your readers what the piece of literature that you are responding to was abouta themean examplea summaryan experience Exponential function f is represented by the table. x -2 -1 0 1 2 f(x) -46 -22 -10 -4 -1 Function g is represented by the equation. Which statement correctly compares the two functions on the interval [-1, 2]? A. Both functions are increasing, but function f increases at a faster average rate. B. Only function f is increasing, but both functions are negative. C. Both functions are increasing, but function g increases at a faster average rate. D. Only function f is increasing, and only function f is negative. A nondiscriminating monopolist:1 will never produce in the output range where marginal revenue is positive.2 will never produce in the output range where demand is inelastic.3 will never produce in the output range where demand is elastic.4 may produce where demand is either elastic or inelastic, depending on the level of production costs Why should women be involved in income generating practictice ? Citric acid (C6H8O7) is an important intermediate in the Krebs cycle and a triprotic acid. What is the normality of a solution made by weighing 128 g of citric acid into a volumetric flask and diluting it to 2 L with water Solid aluminum and oxygen gas react to form solid aluminum oxide. Suppose you have 3.0 mol of Al and 11.0 mol of O2 in a reactor. Suppose as much as possible of the reacts. How much will be left? Round your answer to the nearest 0.1 mol. What is a key distinction between a trial court and an appellate court?A. The presence of a juryOB. The number of witnessesOC. The involvement of attorneysOD. The presence of a judge There are 7 pieces of pie left over in the cafeteria. If 4 pieces are apple and 3are cherry, what fraction of the remaining pieces are apple? Which statement correctly describes the global impact of the American Revolution? Which of these items should not be used if a car is leaking gasoline?Roadside flaresReflective trianglesReflective clothing The function f(x) = 40(0.9)x represents the deer population in a forest x years after it was first studied. What was the deer population when it was first studied?a. 44b.40c. 36d.49 The davidson family wants to expand its rectangular patio, which currently measures 15 ft by 12 ft. they want to extend the length and width the same amount to increase the total area of the patio by 160 ft2. which quadratic equation best models the situation? a = lw (15)(12) (x)(x) = (15)(12) 160 (15x)(12x) = (15)(12) 160 2(15 x) 2(12 x) = (15)(12) 160 (15 x)(12 x) = (15)(12) 160 let p be the perimeter of a rectangle, and let one side of the rectangle be xPLEASE SHOW WORK What does the ball signify in the poem, "The ball poem" by John Berryman?CLASS 10 NCERT Read the paragraph.(1) Dr. Martin Luther King Jr. uses repetition to give his speech rhythm and momentum. (2) "Let freedom ring is a phrase which is used many times in his speech. (3) "Let freedom ring is a phrase that leads to an exciting conclusion.Which revision combines sentence 2 and sentence 3 most effectively?Many times the phrase Let freedom ring is used in his speech because it is an exciting conclusion.Many times the phrase Let freedom ring is used to lead to an exciting conclusion.Let freedom ring is a phrase, and it is used to lead to an exciting conclusion many times.Let freedom ring is a phrase that is used many times to lead to an exciting conclusion.